This site is currently being migrated at a new site. Please read the information below.

LaTeX

Unicode

Showing posts with label real analysis. Show all posts
Showing posts with label real analysis. Show all posts

Thursday, August 27, 2015

Unique fixed point

Let $f:\mathbb{R} \rightarrow \mathbb{R}$ such that $|f(x)-f(y)|\leq k |x-y|$ for some $k \in (0, 1)$ and all $x, y \in \mathbb{R}$. Show that $f$ has a unique fixed point.

Solution

Monday, August 24, 2015

Convergence of improper integral

Let $a \geq 0$ . Prove that the integral $\displaystyle \int_0^\infty\frac{\sin (x^2+ax)}{x}\, {\rm d}x$.

(Qualifying Exams, Wisconsin / Madison University, 2015)

Solution

Sunday, August 23, 2015

Sequence of functions

Give an example of a sequence of functions $f_n:\mathbb{R} \rightarrow \mathbb{R}$ such that each $f_n$

  1.  is continuous in $(-\infty, 0)$
  2.  is discontinuous in $(0, +\infty)$
  3. converges uniformly to a continuous function
Solution

Thursday, August 20, 2015

Definite integral and inequality

Let $f$ be a continuous function on $[a, b]$. If for every $x \in [a, b)$ there exists a $y \in (x, b)$ such that $\displaystyle \int_x^y f(t)\, {\rm d}t>0$ then prove that $\displaystyle \int_a^b f(x)\, {\rm d}x>0$.

Solution

Inequality with integrals

Let $f:[0, 1] \rightarrow \mathbb{R}$ be a differentiable function such that:

${\color{gray} \bullet} \;\; f(0)=0$ and
${\color{gray} \bullet} \;\; 0< f'(x) \leq 1$

Prove that:

$$\int_{0}^{x}f^3(t)\, {\rm d}t\leq \left ( \int_{0}^{x}f(t) \, {\rm d}t\right )^2, \;\;  x \in [0, 1]$$

Solution

Tuesday, August 18, 2015

Vanishing derivative in rational points

Let $f:\mathbb{R} \rightarrow \mathbb{R}$ be a differentiable function such that $f'(x)=0, \;\; \forall x\in \mathbb{Q}$. Prove that $f$ is constant in $\mathbb{R}$.

Solution

Monday, August 17, 2015

Existence of constant

Let $f$ be a non constant function defined on $[a, b]$ and  $f(a)=f(b)=0$. Prove that there exists a $\xi \in (a, b)$ such that:

$$\bigl|{f'(\xi)}\bigr|>\frac{4}{(b-a)^2}\int_{a}^{b}{|f(x)|\,{\rm d}x}$$

 (Qualifying Exams, Wisconsin-Madison, 2015)

Solution

Sunday, August 16, 2015

Convex function

Let $f:\mathbb{R} \rightarrow \mathbb{R}$ be a differentiable, convex and bounded function in $\mathbb{R}$. Prove that $f$ is constant.

Solution

Saturday, August 15, 2015

Existence of constant

Let $f:[0,1] \rightarrow \mathbb{R}$ be a continuous function such that
$$\int_0^1 f(x) \, {\rm d}x = \int_0^1 xf(x) \, {\rm d}x \tag{1}$$

Prove that there exists a $c \in (0,1)$ such that $\displaystyle c f(c) = 2 \int_c^0 f(x) \, {\rm d}x$.

(Duong Viet Thong, Nam Dinh University, Vietnam)

Solution (by Paolo Perfetti)

Wednesday, August 12, 2015

Discontinuous function

Let $f:\mathbb{R} \rightarrow \mathbb{R}$ be defined as:
$$f(x)= \left\{\begin{matrix}
 1&,  &x \in \mathbb{Q} \\
 0&,  & x \in \mathbb{R} \setminus \mathbb{Q}
\end{matrix}\right.$$

Prove that $f$ is discontinuous.

Sunday, August 9, 2015

Functions preserving convergent series

Find all functions $f:\mathbb{R} \rightarrow \mathbb{R}$ that preserve convergent series. (That is the series $\sum \limits_{n=1}^{\infty} f(a_n)$ converges whenever the series $\sum \limits_{n=1}^{\infty} a_n$ converges)

Solution

Monday, August 3, 2015

Rational or irrational?

Let $F(0)=0, \; F(1)=\frac{3}{2}$ and
$$F(n)=\frac{5}{2}F(n-1)-F(n-2), \;\; n \geq 2$$

Examine if the number $\displaystyle \sum_{n=0}^{\infty} \frac{1}{F(2^n)}$ is rational or not.

IMC 2015/ 1st Round/ 3rd problem

Solution

Tuesday, July 21, 2015

Uniformly continuous function

Let $f:[0, +\infty) \rightarrow \mathbb{R}$ be an integrable and uniformly continuous function. Prove that $\displaystyle \lim_{x \rightarrow +\infty} f(x)=0$.

(Berkley Examinations)

Solution

Sunday, July 19, 2015

Zero function from an inequality

Let $f:\mathbb{R} \rightarrow \mathbb{R}$ be a differentiable function such that $|f'(x)| \leq |f(x)|, \;\; \forall x \in \mathbb{R}$ and $f(0)=0$. Show that $f$ is the zero function.

Solution

Zero function

Let $f$ be a continuous function on an interval $[a, b]$. If for every function $g$ on the interval $[a, b]$ such $g(a)=g(b)=0$ and $\displaystyle \int_a^b f(x)g(x)\, {\rm d}x=0$ hold , then prove that $f$ is the zero function.

Solution

Friday, July 17, 2015

Proof of AM - GM inequality using concavity

The AM - GM (arithmetic - geometric mean ) inequality is expressed as follows:

$$\sum_{k=1}^{n}a_k \geq n \sqrt[n]{\prod_{k=1}^{n}a_k} \tag{1}$$

One proof was given by the French mathematician  Augustin Luis Cauchy in its lecture book that he had prepared for his students. The proof is based on induction , the so called "back and forth" form of induction. Since then many proofs of this inequality have been discovered. In this topic we give a proof based on the concavity of the $\log $ function.

Proof

Maximum of a function

Let $f:[0, +\infty) \rightarrow \mathbb{R}$ be a continuous function such that $f(0)=\ell+1 $ and $\displaystyle \lim_{x\rightarrow +\infty} f(x)=\ell$. Prove that $f$ has maximum.

Solution

Thursday, July 16, 2015

Improper integral involving floor function

Evaluate the integral:

$$\mathcal{L}_1=\int_{0}^{\infty}\left \lfloor x \right \rfloor e^{-x}\, {\rm d}x$$

Solution

Integral of floor function

Evaluate the integral:

$$K_n = \int_{-n}^{n} \left \lfloor x \right \rfloor\, {\rm d}x$$

Solution

Tuesday, July 14, 2015

Estimation of series

Let $n \geq 2$. Prove that:

$$\sum_{k=1}^{\infty}\left [ 1- \left ( 1-2^{-k} \right )^n \right ] \simeq \ln n $$

Solution